You are on page 1of 35

Issue Essay 1:

No field of study can advance significantly unless outsiders bring their knowledge and experience to that
field of study.

I strongly agree with the notion that inputs from other fields of study are substantial for the progress of a
particular field. As a matter of fact in this rapidly changing and advanced world no field can prosper on its
own.

I order to establish examples, lets take into account the role of physical and mathematical scientists in the
realm of medical sciences. We all have knowledge that the advance researches in medical sciences field were
only made possible through the advancement in physical and mathematical sciences. Today
physicians/practitioners are able to decipher codes on basic building element of life, i.e. DNA, with the help
of extremely powerful microscopes, which are the product of the research in the field of physics and
mathematics. With the development of MRI machines doctors are now able to see even those injuries or
diseases which were not visible through previous imaging techniques further decreasing the probability of
making an incision through the skin an extremely time consuming endeavor. This has also enabled
physicians to develop more reliable comparison between healthy and morbid organ of human body.

The coordination between Cultural Anthropologists, Geologists, Archeologists, Historians and Biochemists has
open new pathways of study and knowledge before cultural anthropologists. Anthropologists knowledge is
enhanced by the work of Archeologists who unearth ancient remains Geologists who gather information
about rock and mineral formations which eventually lead to the estimation of how old these remains actually
are-, Biochemists and forensic scientists, through hairs, nails and bones of mummified dead bodies can
estimate the populations average life span and cause of death. Cultural anthropologists can also get help
from historians in order to check validity of each other data especially with respect to time scale.

Another striking example of how expertise in diverse fields is needed to advance knowledge involves the area
of astronomy and space exploration. Enhancement in our understanding of solar system, distant galaxies and
universe requires keen advancement in our observation and measurement techniques. Telescope technology
that has enabled astronomers to look and collect data from deep into the heavens is the domain of
astrophysicists. Manned and unmanned exploratory probes are designed by electrical, mechanical and civil
engineers.

In a nutshell, no area of intellectual inquiry can progress without the expert insight endowed by other field
experts. Because sciences are inextricably related and advancement in one field requires a better
understanding of other fields.
Issue Essay 2:

The video camera provides such and accurate and convincing record of contemporary life that it has
become a more important form of documentation than written records.

The given issue asserts that data, acquired by video cameras provide such an accurate and convincing record
that it has become more important than written records.

Well, in many cases this assertion could not hold true. Events processes or procedures having legal value
could not be recorded just by having video films. In every part of the world, to make any action lawful
required some kind of legal documentation to follow.

Consider the example of marriage and issue related to marriage. No legal procedure can involve submission
of marriage video to have marriage registered under law. Obviously a written document would serve as a
proof for a couple to be in married relationship. Same could be imagined for the issue arising for divorce. In
order to get divorce, the concerned parties would have to prove that they had been married in the first place.
Evidently a written document could do better in their regard than showing a video of their marriage
ceremony.

Another case related to marriage complexities is that in some countries it is prohibited for a married couple
to live together without legally bound in a married relationship. Consequently in those countries law dictates
to have a written document as a proof of marriage.

Moreover there could be enormous problems to have video record to legalize a marriage. First of all videos
could be morphed, edited or could be made of any date. This would not be a reliable source upon which
authorities can set their records.

However, video cameras could provide accurate and convincing record in some cases. Lets take an example
of news bulletin. Events of any sort happening through the state or country could be recorded and further
reproduced as healthy evidence. Or in a production factory, quality department can maintain their record by
adhering pictures or video clips to quality control file of a particular project. Manufacturing process could be
recorded and stored to train incoming fresh workmanship in place of having lengthy written procedures
which are significantly cumbersome to read than just watching video.

In case of traffic rules implementation, video cameras had immensely made the task easier. Recorded videos
would suffice to penalize the violators eliminating the need for written documentation altogether.

In sum video cameras do provide essential information which could be recorded, but it could not be
considered to have replaced written documentation.
Argument Essay 1:

"Of the two leading real estate firms in our town---Adams Realty and Fitch Realty---Adams is
clearly superior. Adams has 40 real estate agents. In contrast, Fitch has 25, many of whom
work only part-time. Moreover, Adams' revenue last year was twice as high as that of Fitch,
and included home sales that averaged $168,000, compared to Fitch's $144,000. Homes
listed with Adams sell faster as well: ten years ago, I listed my home with Fitch and it took
more than four months to sell; last year, when I sold another home, I listed it with Adams, and it
took only one month. Thus, if you want to sell your home quickly and at a good price, you should use
Adams."

The author of this argument presented a comparison between two real estate firms and on the basis of
number of real estate agents each firm own, revenue generated and average sale price per house last year,
he concluded that Adams realty is way better than Fitch realty. Finally he made recommendations to choose
Adams realty if anyone want his house or property to be sold quickly and at a good price. I see this conclusion
and recommendation logically unconvincing, as it stands, due to several reasons.

Firstly, the author mentioned that 40 real estate agents were working under the umbrella of Adams realty,
which he deemed as superior as compared to Fitch realty, which has 25 real estate agents and most of them
working part time. I concede partly with this reasoning that, large number of employees working as a part
timer could reflect poorly on organizations profile; however, the author didnt give an indication of the
nature of job of agents working for Adams realty. It is entirely possible that more number of Adams Realtys
agents is working as part timers as compared to Fitch realty. Moreover, one could not be able to comment on
the performance of these two firms merely by considering number of agents working for them. May be Fitch
realty had placed its agents according to market requirements and thus not need as many agents as Adams
realty.

Secondly, categorizing Adams realty as superior could not be substantiated by generated revenue figure. It is
equally likely that, although Fitch realty had generated as low as half of the Adams realtys revenue, the profit
as a percentage of total investment is greater for Fitch realty than that of Adams realty. If the author provides
figures such as profit per dollar invested or profit per employee then it will be a better parameter for me to
judge companys standing. Average sales value is also not a clear indicator of firms superiority. May be,
Adams realty who has higher average home sales value than Fitch realtys got one deal way higher in value
than usual deals during the course of the year which has caused surge in its average sales value. These
average numbers do not answer few questions, like how many deals each firm executed and with what
efficiency? What is the customer response to each firms dealing style and grip on market?

Lastly the author cited his experience as an example to demonstrate the efficiency of Adams realty. He sold
his house, ten years ago, with Fitch realty which took 4 months to finalize the deal on the other hand he
struck a same deal with Adams realty last year and it took that firm one month to finalize. These examples
are from two entirely different time frames and might not present a cogent statistic for comparison.

I need to know the work nature and efficiency of each firms agents. A thorough survey of both the firms
customers should be conducted to quantify customers satisfaction. Figures like profit/dollar invested or
profit/ man-hour of both the firms would present a clear picture on ones superiority. As it stands, I consider
this argument is based on flawed reasoning and wanting in several logical aspects. Therefore, the
recommendations made by the author to choose Adams realty could not be rendered as correct.

Issue Essay 3:

All nations should help support the development of a global university designed to engage students in the
process of solving the world's most persistent social problems.

I agree with the speakers viewpoint that all the nations in the world should make effort to lay foundation of
a university which would eventually take up task to resolve worlds most relentless social issues. To me this
idea would be the most fruitful and productive, if employed efficiently, as compared to any other possible
solution employed to such kind of task.

First of all, founding an educational institution is the most cogent aspect of speakers idea. Educational
institutes are the farthest places from critical social issues like racism, gender discrimination, etc. and are free
from many of the social problems like abortion, birth control, child abuse, etc. Hence, it would be most
feasible for intellectuals, working there, to consider each others view point and present a solution related to
a specific social issue. Moreover naturally such a university shall employ the elite of a particular society thus,
having capability to comprehend and solve problems in the first place. It will also provide a firm ground for
training the youth coming from different parts of the world that would go back to their respective countries
with a practical knowledge of problem solving.

Secondly, having people from different cultural, regional, ethnic and religious attachments could prove
extremely helpful in getting the task done. As, almost all the social issues have different level of bearing on
different societies, living in different parts of the world, opinion from persons from those specific regions
would be indispensable in order to come to a solution to that particular issue. For instance hatred against
underrepresented communities in different parts of the world can be best understood by listening to
accounts from the persons who have experienced that first hand at the same scrutinizing the accounts based
on overstatement can also be done. Such a university would provide a splendid platform for sufferers to
speak, audience to listen and intellectuals to solve having a broad picture in mind regarding those issue.

Finally having presented the above accounts, however, it is quite a daunting task to work out how the things
would be handled at the interface of university and the governments of the nations. For it will remain in the
hand of respective nations governments to decide the legal standing of the solutions proposed by such
university. It is entirely possible that many governments may disqualify propositions by the university in the
name of security of the states ideology or the security of state itself. Also many unprecedented difficult
scenarios may arise from forming such a university. For instance, after extending hands to support the
formation of the global university government of a particular nation may insist to reserve the right to
determine who among the nations diverse classes will have the opportunity to get enrolled into the
university.
In the final analysis, it is always the best way to involve people from different classes in order to come to the
solution for their problems. Intermingling of people related to different backgrounds would make the social
issues much more comprehensible. But to solve intricacies that could be generated, after this university shall
start working, will be the most tiresome task even more tiresome than the formation of university itself. In
my opinion, all the governments must be took into confidence and thoroughly briefed about the working
style and implications of the matters settled at the university.

Issue Essay 4:

"Many of the world's lesser-known languages are being lost as fewer and fewer people speak them. The
governments of countries in which these languages are spoken should act to prevent such languages from
becoming extinct."

I strongly agree with the authors notion up to the extent that those languages cannot survive if their
speakers start to abandon them. In the other part author is considering the responsibility of governments to
take measures in order to preclude the process of languages, being spoken in their countries, becoming non
extant - the part I would dissent a bit to.

Language bears cultural information of the community speaking it. Losing a language can be considered as a
tantamount to losing a culture or the opportunity to studying it. Obviously when nobody would be available
to speak or understand a particular language, how in the world would one study culture associated to it. As a
consequence, a deficit would be created in historical, sociological and cultural studies. So, to save beauty of
cultural diversification of human race we should stop endangered languages going into extinction. However,
this is not entirely a responsibility of the governments of the countries. Even though almost every
government has cultural ministry working under its flag, they cannot take up this task on their own unless a
warning hadnt been escalated by a team of linguists or some NGO working for sole purpose of preventing
languages going into extinction.

Many international organizations like UNESCO and World Oral Literature Project are working to help save
endangered languages. Local linguists can contact these organizations and ask them for support, for they are
better equipped with knowledge and have an infrastructure as compared to that particular countrys
government for solving such a typical issue. Further, after the analysis of the issue these organizations,
working in close collaboration with linguists and people, would suggest some measures or solutions.
Governments role will start there, if any governments help would be needed in order to implement those
solutions. For instance, in India, linguists working in coordination with a team of UNESCO proposed solution
to the government to include an endangered language as a primary education course. Or governments
supportive hand would be required if the solution dictates establishment of new department of that
particular language in local university.

To forestall survival of a particular language becoming threatened is more a matter of altering populations
negative perceptions towards that language. Linguists and local and international social organizations like
NGOs should take up the task of changing negative social connotations towards a particular language, inspire
interest within the affected community and inspire interest among linguists rather than considering
government as a sole responsible for the task, which certainly had more important things to care about than
to save a language. An environment should be created within a community to encourage bilingualism
amongst the people. Programs, seminars and conferences should be held to throw light on literary work in a
particular language, inspire others to pursue a career in literature of lesser known languages and to remove
prejudices and hatred among people against any language.

In summary, it is a great loss of humanity and several areas of knowledge if some languages go into extinction
and this is plausible as technological revolution is changing world into global village and more and more
people tend towards one single language. And it is the responsibility of the whole community and especially
of linguists to devise and organize programs such that endangered languages can be saved from dying out. At
the end governments role can come into play, only if required, to help support implementation of such
programs in any aspect.

Argument Essay 2:

The following was written as a part of an application for a small business loan by a group of developers in
the city of Monroe.
"A jazz music club in Monroe would be a tremendously profitable enterprise. Currently, the nearest jazz
club is 65 miles away; thus, our proposed club, the C Note, would have the local market all to itself. Plus,
jazz is extremely popular in Monroe: over 100,000 people attended Monroe's jazz festival last summer,
several well-known jazz musicians live in Monroe, and the highest-rated radio program in Monroe is 'Jazz
Nightly,' which airs every weeknight. Finally, a nationwide study indicates that the typical jazz fan spends
close to $1,000 per year on jazz entertainment. It is clear that the C Note cannot help but make money."

Write a response in which you discuss what specific evidence is needed to evaluate the
argument and explain how the evidence would weaken or strengthen the argument.

The above argument in which author had concluded that his new business endeavor would be a lucrative
project is based on flawed assumptions, insufficient evidence and non-convincing premise. Author had taken
into account several reasoning from unavailability of another jazz music club, which author is interested in to
start, to popularity of this particular style of music among residents of the city, however, lacking in several
aspects.

Firstly, the author started off with an intermediate conclusion that, as no other jazz music club is operating
within the vicinity of the city, nearest to be 65 miles away, his music club would take up whole local market.
Here, author didnt mention the reason behind non-operation of any other jazz music club or availability of
other forms of music clubs within or nearby the city. Maybe residents of Monroe do not have any specific
interest in music thats why are content with the available music form in the city. Moreover, Jazz is a unique
cult in musics versatile family which mostly is attractive to young generation. A thorough survey needs to be
conducted incorporating every age class of the citys residents in order to substantiate that what percentage
of Monroes resident do actually had an inclination towards jazz music.

Secondly, author mentioned that over 100,000 people attended last jazz concert but didnt give total
population figure. Maybe people attended last jazz concert were a meager proportion of the total
population. Moreover one cannot base an argument upon number of people attending a festival for it is likely
that many people attending a festival for just recreational purpose having no actual interest in what the
festival is about. As, several jazz musicians are living in Monroe but no one was struck by the idea of opening
a club in the city indicates towards weak commercial aspect of the endeavor. Listening to radio is not a
popular trend among masses these days. Maybe among a few people who listen to radio regularly the
highest proportion is of jazz music listeners, because they have no other place to look for their typical interest
and thats why Jazz nightly become the most listened program on radio.
Thirdly, author quotes a nation-wide statistic and converge it on a city to show expenditure of a typical jazz
music fan. This does not look like a cogent reason. There is no evidence to believe that this statement could
be considered true for the resident especially fans of jazz music living in Monroe. It is entirely possible that
jazz music fans in Monroe only have a liking for jazz music up to the extent of, merely, listening and they
wouldnt want to expend anything for their music liking. Even if they were willing to expend their money on
listening music then the question would come to their number. How many fans actually want to spend
something on listening jazz music and how much?

In the final analysis, in order to arrive at a conclusion a thorough survey needs to be conducted to have the
opinion of the people regarding their music liking and views on available music options in the city and their
willingness to spend on listening jazz music.

Issue Essay 5:

To understand the most important characteristics of the society, one must study its major cities.

Write a response in which you discuss the extent to which you agree or disagree with the
statement and explain your reasoning for the position you take. In developing and
supporting your position, you should consider ways in which the statement might or might
not hold true and explain how these considerations shape your position.

The above statement claims that to be able to discern the most significant traits of a society, one has to study
its major cities. We all know this demonstrably, that as means to comprehend significant traits of a society or
nation, its history, culture, traditions, language, religion, politics, arts and music need to be contemplated.
Greatest source of these knowledge areas are major cities for they are the places which had seen several
courses throughout their history, they have imprints of the stories of hardships, pleasures, failures and
triumphs of people living among them and traceable memory of new cultural, traditional, lingual or arts
attachment in their DNA.

We always tend to study big cities as one of them certainly has been the official seat of the monarchs, the
history of that city has seen. Religion, cultural priorities and policies of the rulers leave an everlasting
impression on the fabric of societys characteristics. Rulers who maintained a policy of jingoist culture raised
more war veterans as compared to those with enlightened pragmatic outlook. Later shall have a history of
breeding great scientists or artists.

Most major cities bear a timeline of history in which accounts of renowned travelers coming and staying in
those cities, foreign armies assailing, in coming masses from remote areas due to the threat of contagious
disease are mentioned. These accounts render cities a better canvas that can demonstrate cultural richness,
cherished traditions, political and religion aspirations through historical monuments, architecture and
educational institutes of the nation. Authors and tourists travelling for educational purposes could get benefit
from these accounts to develop a perception of the characteristics of the people living in that society.

It is also true that rural or remote areas do share some aspects of nations traditions, culture and history or in
few cases can be the best representative of societys multi-faceted characteristics. Studies of prehistoric
times require archeologists to search for ancient remains which are not very likely to be found in the middle
of the citys dark and narrow streets. Moreover, in the modern age of globalization and urbanization more
and more people tend to flow toward cities to prosper and raise standard of living, as, almost all the
ingredients required to accomplish this grueling task of prosperity are readily available there, big cities
became more an amalgam of cultures and traditions which these incoming people bring with them.

As a matter of course, society can discard or abandon outdated practices, not conforming to the standards of
this age of technological revolution and may take up the practices of incoming masses. This situation could
behave as a deceiving feature to the person studying, in its big cities, nations pure characteristics which
might have got lost in diversity.

In a nutshell, cities do provide valuable information on societys important features. Anybody who wants an
understanding of societys characteristics needs to study all the important aspects, of the cities, which define
human life. But the significance of rural part could not be overlooked as the markings of values lost in the
humdrum of the cities, as a consequence of this fast age, could be found out there.

Issue Essay 6:

Educational institutions have a responsibility to dissuade students from pursuing fields of study in which
they are unlikely to succeed.

Write a response in which you discuss the extent to which you agree or disagree with
the claim. In developing and supporting your position, be sure to address the most
compelling reasons and/or examples that could be used to challenge your position.

What good an educational institution is, if it cant help its students to choose a career path? This is not only a
fundamental question; we usually aim at our education providing centers, but also this can serve as a
performance indicator for the same. Educational institutions are liable to guide its pupils, must go up to the
length of persuasion and dissuasion and must be held accountable for the effects of their decisions.

Organs of the educational institutions i.e. teachers, career counselors, physical training instructors, sports
coaches and principals are more familiar persons to students natural propensity, mental and physical
abilities and difficulties, which students are facing. Furthermore, these drivers of educational organizations
are more equipped with knowledge in their specific fields. They are, therefore, logically more suitable to
deter students to chase fields of study inappropriate for them. As, a science teacher can evaluate each of his
pupils abilities, so a particular sports coach or literature teacher, and can propose convincingly that whether
or not a specific student can pursue that specific field as his/her career field. It is true that most people start
concerning about their careers in the high school, however, this practice would be most fruitful if it starts off
from schools.

Having said and believing that, we cannot put responsibility on the students to choose field of study on their
own, which is a potentially risky task, leads us to ask a fundamental question. How can we build an air of trust
on mere instructors recommendation about someones future, for they are also vulnerable to human
weaknesses such as biasedness, resentment, incapability etc.? As an answer to this and to judge carefully
Teachers/Instructors recommendations about a particular students true potential, a robust students
evaluation system should be formed. This system should take into account real time performance based on
exams reports, personal aptitude reports, each teachers and career counselors rating interpersonal skills
evaluation and physical fitness or particular sports skills rating of the student for all the past years. Schools
may get help from psychologists to underscore students natural penchant. This data would help quantifying
cognitive, emotional and physical traits of the students. In this way components of educational institutions
would build their credibility to effect students decisions regarding their future.
Issue Essay 7:

Scandals are useful because they focus our attention on problems in ways that no speaker or reformer ever
could.

Write a response in which you discuss the extent to which you agree or disagree with
the claim. In developing and supporting your position, be sure to address the most
compelling reasons and/or examples that could be used to challenge your position.

The above statement tends to draw our attention towards a positive use of extremely negative, maybe in
view of most of us, phenomenon. Such a notion helps us to identify loop holes and short comings in our
domestic, social, and political systems. Is there anything more helpful than a practical scenario to help
diagnose, address and rectify minute hidden flaws in theoretical policies and dogmas?

We usually refer the term scandal to some wrong doing of a well-known figure; let it be politicians,
sportspersons, film stars, etc. Political scandals generally point out loop holes in policies and laws, made
under the supervision of some power possessing people of the society, which may or may not be left
deliberately and to make use in the time of requirement. In this way public and other faction of democratic
system can raise the matter to address such flaws in law. Courts, which are responsible to elaborate laws for
implementation, can revisit and revitalize its elaboration. In this way a sense of knowing could be generated
among common masses.

Lets take an example of some figure that has no attachment to the countrys political system. When their
scandals surface, people related their profession can take it as a lesson which can help them reshape their
characters and pay extra attention to minute details to move in public.
Scandals can serve as guidance to general public and new comers of some specific field. They are most
importantly lesson learning (left Incomplete).

Argument Essay 3:

"Woven baskets characterized by a particular distinctive pattern have previously been found only in the
immediate vicinity of the prehistoric village of Palea and therefore were believed to have been made only
by the Palean people. Recently, however, archeologist discovered such a Palean" basket in Lithos, an
ancient village across the Brim River from Palea. The Brim River is very deep and broad, and so the ancient
Paleans could have crossed it only by boat, and no Palean boats have been found. Thus it follows that the
so-called Palean baskets were not uniquely Palean.

Write a response in which you discuss what specific evidence is needed to evaluate the
argument and explain how the evidence would weaken or strengthen the argument.

The author of the argument concluded in denying the previously established fact of a unique character of
particular distinctive pattern baskets. He based his conclusion on the reasoning that as there are no obvious
means to transport these baskets to other village and the basket was found in the ancient remains of
another village across a deep river. So, the basket was not a unique product of former village. I see this
reasoning as insufficient and obviously other evidences are required to establish such a conclusion firmly.
First of all, archeologists should search for specific tools that could be used to make these baskets in both
the villages. If such tools, which archeologists had considered to be used in making the baskets, were found,
then such evidence would strengthen the argument. Otherwise it will be a sign which would render these
distinctive baskets, a Palean native.

Secondly, it is explicitly mentioned in the argument that no sign of Palean boats were found in the remains
of the city, though, itll be equally important to look out for signs of Lithosean boats. Maybe people pf
Lithos village would have made a boat and had been using it for trade purposes with other villages. Any sign
of Lithosean boats would weaken the argument.

Thirdly, it would be important in my view to also search for trade patterns of that area. Maybe Palean
village had been trading with other villages adjacent to it and those villages had been trading these baskets
with Lithos village. They also would need some evidence of boats in those villages as well. It is entirely
possible that Palean people had trade relation with the people of village adjacent to theirs and those, other
village people had been trading baskets with Lithos village. I such case any sign of baskets and boat in the
villages adjacent to Palea would weaken the argument.

Lastly, it is also not clear from the above argument that whether any sign of boat itself was not found or
apparatus and manufacturing facilities kind of thing was not found in the village of Palea. Author should
investigate and mention clearly that on what bases is he saying that ancient people of Palea were not
capable of making boats? If there is any sign of tools that can be used in making boats or huge work places
indicating the manufacturing of boats are there, then it could be inferred, that Palean people were making
boats and obviously using them for trade purposes. I such a case it would greatly weaken the argument.

My final words are, unless the author presents acceptable answers to above raised questions his conclusion
is logically lacking. Therefore, his argument is not convincing.

Issue Essay 8:

Claim: Governments must ensure that their major cities receive the financial support they need in order to
thrive.
Reason: It is primarily in cities that a nation's cultural traditions are preserved and generated.

Write a response in which you discuss the extent to which you agree or disagree
with the claim and the reasons on which that claim is based.

It can be easily conceived that cities are any nations cultural hubs. It is primarily a Governments
responsibility to help support and preserve delicate aspects of nations traditions through which its culture is
breathing.

Admittedly all the main aspects of any nations diverse cultural spectrum do flourish in cities. This is due to
the fact that large cities generally have rich historical background, historical buildings and monuments, most
refined dialect of language, and have in its core its literary richness and religious attachment of major ethnic
groups of the society. As a result of this cultural diversification, major cities of the countries are the main
attention catchers of the outsiders. Outsiders visit these cities due to a number of factors. So, in order to
maintain cultural core values governments must provide financial assistant to those departments who are
taking care of such issue.
Any citys progress can be measured from viewing betterment of its people. It is a well-known fact that any
countrys major proportion of population live in cities. As a matter of course, major cities are rightfully in
more need for financial support to accommodate ever increasing influx of people than place like country
sides or villages. One can easily come to a point that this increase in population of the cities put its cultural
entities risk. For instance, because of surge in urbanization it is possible that countrys main historical
buildings and architecture may lose its charm due to negligence of government on their part, which can in
turn result in fiscal deficit due to lack in tourism.

Countrys other parameters to its cultural values are education, food, traditional agricultural or industrial
products and environment also need special government attention for their progress. It is quite a general
practice for governments to pay heed to its educational institutions but from the point of view of preserving
culture in rare. For example existing campus of citys famous college or university that depicts a specific
ancient architecture which is now may be very uneconomical for government to follow who has a plan to
extend its campus. So, to keep that architecture intact government has to place an increased budget.

All the above discussion does not in any case means that villages need no special attention from government
or any of other responsible organization. It is also in the knowledge of most of us that villages depict some of
the deep rooted traditions, some domestic industries and language. In many cases these values are in their
purest form in villages away from the eyes of major population sector. Government may be not able to take
care of each and every place on its own for there are many other pressing issues present, such as
unemployment, poverty, security, etc. Therefore, local and international organizations should come forward
to help preserve cultural diversity of remote areas.

In the final analysis, for cultural preservation, governments financial support is indispensable. But, for
remote areas it has a little to do with government for it can be taken care by local international bodies who
are out to serve this purpose.

Argument Essay 4:

"A recent study of eighteen rhesus monkeys provides clues as to the effects of birth order on an
individuals level of stimulation. The study showed that in stimulating situations (such as encounter with
an unfamiliar monkey), firstborn infant monkeys produce up to twice as much of the hormone cortisol,
which primes the body for increased activity levels, as do their younger siblings. First born humans also
produce relatively high levels of cortisol in stimulating situations (such as the return of a parent after an
absence). The study also found that during pregnancy first-time mother monkeys had higher levels of
cortisol than did those who had had several offspring.

Write a response in which you discuss one or more alternative explanations that could rival the
proposed explanation(s) and explain how your explanation(s) can plausibly account for the facts
presented in the argument.

Author presented an explanation to cater difference in levels of stimulation of different animals. While it can
be considered as true that birth number can render many, divergent from normal, aspects to an organism,
however, as a matter of fact, we cannot say for sure which factor is the most contributing to those divergent
characteristics unless we incise every possible explanation with a critical eye. Therefore, the above argument
also requires an alternate explanation.
First of all, author has mentioned that just eighteen monkeys of some specific family name were studied
which is itself insufficient to make it a base of any conclusion. In order to gain more credibility this study
should include a broader range of families of different organisms, if not every.

It is entirely possible, for that specific monkey group (rhesus, in this case), to be an inhabitant of such an
environment in which first born depicts higher levels of stimulation than normal. Maybe their community
system is such that first born are specially taken care of as compared to their younger siblings. This special
attention and been fed on special kind of food renders an explanation to increased level of certain hormones.

Author has widened the scope of study to explain higher level of stimulation to humans, though, without
stipulating any supporting fact. One could not extract real time meanings from, just, as vague terms as
high. This proposition begs a question. How much high? How many individuals were studied for this effect?
The above mentioned explanation can carry humans, as well, in its domain. Because humans also portrays an
inclination to first born as compared to their other off springs. First born are usually brought up under the
umbrella of special affection and care. It is a widely known fact that, we humans develop most of our
personality traits at younger age and our all organs and systems responsible for controlling and generating
hormones show imprints of the environment in which we all are brought up.

First born also depicts some other personality traits more pronounced than others e.g. higher sense of
responsibility, intelligence, enhanced levels of adaptability and these all boils down to changes in hormone
levels. What is the point in saying about all these changes if not to contemplate upon the environment in
which they are raised? Surely and definitely, the kind of attention which a child gathers at an early stage
stimulates its organs to function properly, which is reflected as a divergent property in his adulthood.

So, we can conclude that may be it is not an innate ability of first born to show an enhanced level of
stimulation. Any organism can show same level of hormone generation, which results in, more energetic,
more responsive and passionate organism, if given same attention, care, love and confidence.

Issue Essay 9:

Some people believe that government funding of the arts is necessary to ensure that the arts can flourish
and be available to all people. Others believe that government funding of the arts threatens the integrity
of the arts.

Write a response in which you discuss which view more closely aligns with your
own position and explain your reasoning for the position you take. In developing
and supporting your position, you should address both of the views presented.

In the first opinion, speaker has related government funding to the progress of arts and its availability on
equitable basis to general public. Availability of sufficient funds is an essential ingredient, for any field to
thrive, nevertheless, the author fails to consider several important facts. In the second opinion, the author
displays a fear that by subsidizing arts, government is very much likely to damage its essence, the part I
would agree with the most.

Firstly, financing arts is not a proper responsibility of the government. Obviously, there are a lot attention
seeking and pressing issues in the realm of government like unemployment, poverty, security, public health,
etc. prosperity of arts would be nowhere in the vicinity of the priorities of the government. Affluent private
sector is there to fill this void. From the past few centuries, industrial magnates had been subsidizing artists.
Resourceful production houses can help music industry to develop and flourish. Since, there is a well adopted
system in place to finance arts of a particular region; this will be a time and resource wasting endeavor to pull
government in such matters.

Governments help might be required in the exceptional case of non-availability of funding, but that too
couldnt be adopted as the only option. In this age of information explosion, media houses and I. T. firms
could take up the task of preserving arts.

Also, governmental setups such as cultural ministry, etc. are marked with incompetency and inadequate
resources to handle such tasks effectively and thus, cannot be regarded as evenhanded patron of arts and
culture. Restrictions, priorities and choices, as they are the signature of governmental setups, put them in a
place, untrustworthy to the elite of artists community. Furthermore, it is entirely possible for legislators to
have narrative about arts which could be considered as misguided or unrepresentative of the general
populace. Or maybe governmental funding is more prone to serve legislators or other lobbyists purposes
and is less likely to fulfil its literal purpose.

In the final analysis, as, philanthropy is alive and well today government need not intervene to ensure that
our arts and culture are preserved and promoted. Moreover, government support to such ends would do
more damage than good due to numerous ditches present in the way of governments working.

Argument Essay 5:

The following appeared as a letter to the editor from Central Plaza store owner.

"Over the past two years, the number of shoppers in Central Plaza has been steadily decreasing while the
popularity of skateboarding has increased dramatically. Many Central plaza store owners believe that the
decrease in their business is due to the number of skateboard users in the plaza. There has also been a
dramatic increase in the amount of litter and vandalism throughout the plaza. Thus, we recommend that
the city prohibit skateboarding in Central Plaza. If skateboarding is prohibited here, we predict that
business in Central Plaza will return to its previously high levels."

Write a response in which you discuss what questions would need to be answered in order to
decide whether the recommendation is likely to have the predicted result. Be sure to explain
how the answers to these questions would help to evaluate the recommendation.

The speaker has made recommendation based on some activities like skateboarding, going on in the
plaza and considers them as the sole reason for decrease in business level of the plaza. In laying such
argument the speaker fails to answer some of the compelling question that may arise while keeping in
view such an exacerbated situation.

Firstly, it is a well-known fact that progress of a particular business depends upon competition and
market conditions. A valid question that could be asked is, had the decrease in the number of shop
owners in Central Plazas caused due to operational upper hand of some other plaza in the nearby
vicinity of Central Plaza or within the city? Has the customers inclination changed to some other area? If
the answer to both of these queries is in positive then it has nothing to do with the skateboarders.
Central Plaza store owners need to consider ways to revive business in their vicinity.
What is the quality of goods being sold at Central Plaza? Maybe store keepers in Central Plaza remained
ignorant about the shift in manufacturing sector and therefore, still relying on old fashioned goods. As a
result customers attention has been shifted towards other commercial area which could be more in line
with modern trends related to particular good. For example we see a constant shift in quality in sports
goods with the progress of research in this field.

As, the speaker pointed out, accumulation of trash and surge in hooliganism in the plaza, are the clear
indicators of carelessness from the part of plazas administration. Is there any bar or small casino
opened within last few months? Central Plazas administration has to take this issue very seriously. Or,
the store keepers are the ones doing such activities inside the plaza. Obviously, no one would want to go
to such a place for shopping. Well, in this case shop keepers themselves do not want their business to
prosper. Even if the city prohibits skateboarding that would cause no effect on the sulking business of
Central Plaza.

It is also true that some people make abuse of some activities or some societies see some activities with
suspicious eye, nevertheless, it should not mean that those activities should be banned. Obviously,
banning and prohibition are extreme steps and should be taken after close and precise evaluation of the
activity. We all are well aware of the fact that skate boarding is a healthy activity and one could not
compel the residents of a city, to view it as a cause of lowering of business of certain plaza.

In the final analysis, we can evaluate the logical standing of authors recommendation upon the answer
he would provide to above stated questions. Therefore, I see the reasoning upon which the author
based his recommendations illogical and unconvincing, as it stands.

Issue Essay 10:

In any field of endeavor, it is impossible to make a significant contribution without first being strongly
influenced by past achievements within that field.

Write a response in which you discuss the extent to which you agree or disagree
with the statement and explain your reasoning for the position you take. In
developing and supporting your position, you should consider ways in which the
statement might or might not hold true and explain how these considerations shape
your position.

According to the author, in order to make the most valuable achievement in any enterprise, the principal
requirement is to get truly inspired by accomplishments made in the past. Admittedly, one can get
inspired through past achievements; however, author fails to consider several important factors which
should be taken into account while evaluating the reasons behind those achievements.

Firstly, if the attainments of ascendants were the only way possible to inspire an individual to achieve
how one could explain the driving force behind accomplishments of prehistoric people, when there was
purportedly no foundation of knowledge laid. Inherent ability of inquisition of mankind comes to ones
mind here as an explanation. Probing nature, propensity to understand phenomenon in their originality
and striving for the continuous improvement are the most fundamental characteristics of human
species, which inspires any pursuer to start off and keep going.
A lot is happening around us, which could be sufficient to instigate the urge of knowing more and to
stand out. An apparently banal instance of an apple falling could carve out an unparalleled genius.
Observing and questioning, arms our mind with the power of developing analogies which renders one a
different perspective of looking at a problem. Thus, it is the intrinsic quality of human mind to ponder
over the processes taking place around us.

How can this tremendous surge in development, which we see in the world today, be possible if
everyone had sought to see through the spectacles of their ascendants or peers? Obviously, a lot more
happened, in any field, due to its pursuers stance to renounce established norms. Existing knowledge
could be beneficial but up to a limited extent. One can quote numerous examples in which one can
render credit of solving the most pressing problems, to solvers courage to break with traditions.
Furthermore, geniuses, who consider themselves as a problem solver often, tend to move in the
opposite direction.

On the other hand, a milieu, appreciative of probing nature and rife with anecdotes of geniuses striving
for truth and perfection could act as catalyst for multitudes to strive further. As one of the genius
described himself as I could only see an ocean of knowledge by standing on the shoulders of giants.
Our ascendants past achievements could serve to ignite a spark in us. Thus, we can prepare ourselves to
dive into that ocean keeping our own way of thinking intact.

In the final analysis, past accomplishments can serve to stir up an individual to attain his/her dream,
though, mankinds intrinsic ability to look deeply into the phenomenon, the urge to thrive and tendency
to shatter traditions, if they are obstructing ones travail, ought to be fundamental in its achieving the
unachievable.

Argument Essay 6:

The following appeared in a letter to the editor of Balmer Island Gazette.

On Balmer Island, where mopeds serve as a popular form of transportation, the population increases to
100,000 during the summer months. To reduce number of accidents involving mopeds and pedestrians, the
town council of Balmer Island should limit the number of mopeds rented by the islands moped rental
companies from 50 per day to 25 per day during the summer season. By limiting the number of rentals, the
town council will attain the 50 percent annual reduction in moped accidents that was achieved last year on
the neighboring island of Seaville, when Seavilles town council enforced similar limits on moped rentals.

Write a response in which you discuss what questions would need to be answered in order to
decide whether the recommendation is likely to have the predicted result. Be sure to explain
how the answers to these questions would help to evaluate the recommendation.

While limiting the number of vehicles running on road, especially during vacation season, might help
curtailing the number of accidents occurring on roads, it cannot be considered as a sole factor to this
task. The authors recommendation is begging the question and without satisfactory explanation to
those questions, I would consider the above argument as logically unconvincing.

Firstly, the author started off with quoting a population figure but didnt mention in which age group the
major section of the population spectrum lies. Maybe, the major portion of this population comprised of
children, at or below age of 5 or above age of 60. In both the cases the biggest constituent of the
population is unlikely to gather in huge numbers at recreational places that would otherwise be very
likely if this major portion had comprised of youth, between 16 to 30 years of age. So, if this is the case,
there are low chances of people gathering in recreational places or in island center, areas more prone to
accidents generally, and could be translated in the low chances of accidents occurring during summer.

Secondly, the author hasnt mentioned kind of other vehicles running on the roads of Balmer Island.
Maybe more number of accidents had been occurred due to other forms of transportation, over the past
few years, as compared to the most popular mopeds. Furthermore, the author hasnt said anything
about the nature of accidents taken place on Balmer Island over the past few years. What percentage of
total accidents has occurred because of mopeds? If that percentage is minute then cutting mopeds
number, roaming on the roads, would have no effect on overall statistics of accidents. As the matter of
course accidents should be categorized according to the place of their happening. Like, what percentage
has happened in the resident areas or near the recreational places? As, these regions would be more
likely to bear movement of mopeds, low percentage of accidents in these regions, would not call for
curbing the mopeds movement as a safety precaution.

Lastly, the author attempted to substantiate his argument by quoting an example from neighboring
island Seaville. It might be true that neighboring islands authorities had implemented laws that curbed
the mopeds movement but the author hasnt said anything that had only the mopeds movement
curtailed or all the means of transports curtailed, during the past year in the neighboring island? If all the
forms of transportation were reduced in number then it would not substantiate the authors
recommendation. It is equally likely that to reinforce curtailment rules further and to reap maximum
benefit from them, authorities on the neighboring island had also passed strict rules for the observance
of traffic rules, like not allowing high speed vehicles near the vicinity of recreational places during peak
hours, charging heavy monetary amounts on over-speeding, etc. In all these cases just curbing mopeds
rental issuance would not serve the purpose as it had served in neighboring island.

In the nut shell, without giving satisfactory answers to above mentioned queries, I consider the premise
upon which the author based his recommendation as logically flawed and therefore, unconvincing as it
stands.

Issue Essay 11:

Claim: In any field business, politics, education, and government those in power should step down after
five years.
Reason: The surest path to success for an enterprise is revitalization through new leadership.

Write a response in which you discuss the extent to which you agree or disagree
with the claim and the reasons on which that claim is based.

Often changings bring out the best amongst us. For a nation, party, organization or team, in order to
meet the pace of developed world or to revive lost glory, doing alterations in the leadership, often
times, becomes inevitable. However, clarity in achieving goals and honesty in pursuing progress of the
selectors would play a key role in deciding the outcome of these changes and if the selection mass is
deprived of these qualities, the change process could become more harmful than good.

All the time we observe changes in our surrounding. This property has its mark deep into the fabric of
nature. Intrinsically change is directed to bring about something good. In the modern world, most
nations have resorted to adopt democratic political and governmental system, which is based on very
philosophy of changing leadership from time to time. If the monarchy had, indeed, so successful then
the people would not have rejected that. The roots of this choice lie in the fact that not trying new faces
in any system could prove to be a grave mistake. Furthermore, the world has seen that it is very likely
that the monarchs, who mostly rule without the fear of being replaced, are more vulnerable to
corruption, oppression, nepotism and biases than elected governments which govern for a limited time
and are liable to answer for their performance. The very feeling of being replaceable will keep rulers
from getting their hands dirty.

Secondly, changing process is analogous to render others with opportunity to perform. And it could
serve to choose for the fittest and the most apt amongst us to do a job. As a matter of course, those who
get replaced will try hard to overcome their short comings and weaknesses in order to reclaim their
leadership, thus, creating an air of positive competition which is in turn tremendously beneficial for the
system. A colleges or universitys former department head would revisit his shortcomings and strive to
overcome them and could reclaim his office as a more enlightened and progressive leader.

Performance and prosperity of an organization or a sports team is directly related to the understanding
between their management and the team members. Periodically changing leadership would most likely
provide the organization with the best combination depending upon employee-management
understanding. It is also not far from possible that this understanding could any time deteriorate and
threatens the very survival of the organization. Thats why systematic periodic refreshment of leadership
is important. Also the changes in management would ease those employees who were the victims of
previous managements resentments.

Lastly, to reap maximum benefit from the changeableness of the leadership the selecting mass should
be the most knowing ones. Sometimes the biases of selecting entities jeopardizes the very purpose, this
change is intended to fulfil. Also the people who are bringing about the change must not be prone to be
made fool. Changes brought by gullible or amenable masses, which are most likely to be moved by mere
rhetoric, might be proved to be nemesis for an already running system no matter, however the grave
the situation is. What good the change is if it exposes an efficiently working system to the entire
vulnerabilities one could ever imagine of. Therefore, a mere change in the system would not do any
good if its not coupled self-correcting and self-learning streams.

In sum, in almost all the cases, changes are intended to be made out of the sense of betterment and
prosperity. But the possibility of reaping the intended outcomes greatly depends upon the intentions,
acuteness and pragmatism of critical thinking of the people bringing about the change.

Issue Essay 12:

Nations should pass laws to preserve any remaining wilderness areas in their natural state, even if
these areas could be developed for economic gains.

Write a response in which you discuss your views on the policy and explain your
reasoning for the position you take. In developing and supporting your position, you
should consider the possible consequences of implementing the policy and explain how
these consequences shape your position.
Preserving and protecting environment should not only be considered to serve our benefits but also its
our moral responsibility. No financial sake, however the lucrative may be can worth a price of
endangering natural environment. Nevertheless, under inevitable circumstances environment should
not be harmed up to the extent that it loses its assimilative capacity.

All living organisms are dependent upon their surrounding for their survival. Quality of air that we
breathe, water we drink, soil or land from which we take all forms of our food are dependent upon our
activities. The more we behave in a responsible manner towards our environment the more we would
be liable to reap benefits from what our environment has to offer. Like its totally upon our disposal that
either we cut down the whole woody patch for an ambitious production facility or housing society or
leave the natural land in its place and use a comparatively barren land for business purposes. Choosing
the latter option may result in nary to low loss in profit in business terms but consequences of former
would certainly not turn out so well.

Putting environments natural processes in peril for the sake of monetary gains is not a perfect make out
considering the stakes involved. Very often areas of wilderness are home to versatile species and
represent characteristic flora and fauna of a particular region. Due to these species ecological cycle of
the region is running. Any disturbance in the natural habitat would translate into endangering the very
existence of any particular specie and thus, disturbing the whole ecological cycle. One can quote many
instances in which governments have ordered to wind up hugely valuable and lucrative facilities just to
preserve natural habitats of particular species. We as a human being, the most intelligent of all species,
has this responsibility on our shoulders to protect, in all means, the lives and habitats of other living
organisms. For we are getting immense benefits from the role these organisms are playing in sustenance
of our environment.

In none capacity I meant that infrastructural and technological development should halt in the name of
environmental protection and preservation. Our environment has a capacity to protect itself and
compensate the disturbances being indulged by activities of organisms living in it. But there is a limit to
this self-preservation phenomenon. A lake that is responsible for sustaining and supporting natural
habitat around it would not compensate for the whole citys waste, if being thrown into it. Likewise, one
can study, how much of the total area of the forest might be cut down in order to get vacant land for
development of a society that would not affect the natural habitat and ecological cycle of the species
living among that forest. Nowadays, almost everywhere its a common practice to have
environmentalists as a consultant to study, recommend and approve ways to execute developmental
works keeping the natural environment intact.

In summary, areas of wilderness are of extreme importance to health, well-being and sometimes to the
very existence of many living organisms including us. As a matter of course, we cannot imagine sound
and happy human race without help preserving it in its very natural state. Moreover, as the most
intelligent being we bear a responsibility on our shoulders more than anyone on this planet to protect
natural environment. A responsible and thoughtful employment of our ambitions would serve to keep
this planets congeniality intact.

Argument Essay 7:

Arctic deer live on islands in Canadas arctic regions. They search for food by moving over ice from island to
island during the course of the year. Their habitat is limited to areas warm enough to sustain the plants on
which they feed and cold enough, at least some of the year, for the ice to cover the sea separating the
islands, allowing the deer to travel over it. Unfortunately, according to reports from local hunters, the deer
populations are declining. Since these reports coincide with the recent global warming trends that have
caused the sea ice to melt, we can conclude that the purported decline in deer populations is the result of
the deers being unable to follow their age-old migration patterns across the frozen sea.

Write a response in which you discuss what specific evidence is needed to evaluate the
argument and explain how the evidence would weaken or strengthen the argument.

The author in the above argument has made relation between diminishing populations of deer in the
arctic region with global warming, which is not a cogent result for one to arrive at, and is rife with holes.
In order to evaluate the argument evidences would be required and in absence of these evidences, I
would consider authors inference logically unconvincing, as it stands.

Firstly, the author based his argument from just the mere sayings from local hunters who are without
doubt not trained to comment on such issues and their observations may not be deemed as reliable as
compared to someone professional in this field. It is entirely possible that due to scarcity of food deer
population has moved to some other area and hunters cannot locate them in huge numbers as they
used to do previously. An elaborate statistical data from environmentalists and zoologists, studying the
arctic region, would present the better picture of effects of global warming on the populations of the
species inhabiting arctic region.

Secondly, the author also hasnt mentioned the effect of global warming on other animals inhabiting
arctic region. Maybe, deer have made them compatible to such changes in temperatures and a large
portion of their populations has moved to regions where sufficient food is available during winter
season. An elaborate study is required on the effects of global warming on flora and fauna of arctic
region to evaluate the authors inference.

Thirdly, the author has mentioned that due to melting of ice deer populations could not follow the path
used by their ancestors. May be alterations in environmental patterns over the years have compelled
deer populations to renounce that path and search for new ones, especially those leading to areas rife
with vegetation. The author needs to provide evidence and study stressing on the effects of leaving old-
age pathways on deer populations. Moreover, evidence is required to rule out possibility of deer
populations adopting some other area for migration.

In sum, I would ask for the statistical data from the professional persons on the scarcity of deer
population in the arctic region. An elaborate study evaluating the effects of global warming on the whole
living species of region, especially on deer populations is required to evaluate the arguments validity. In
absence of these documental and statistical proves I would consider the argument unpersuasive and
logically wanting, as it stands.

Issue Essay 13:

Peoples behavior is largely determined by forces not of their own making.

Write a response in which you discuss the extent to which you agree or disagree
with the statement and explain your reasoning for the position you take. In
developing and supporting your position, you should consider ways in which the
statement might or might not hold true and explain how these considerations shape
your position.

Behavior is an intrinsic property, expressed through our actions. In other words, it is our behavior that
determines that perception of ours we render to the outer world. Admittedly, all of us are entangled to
an innate humanistic nature, perhaps some of us are fortunate enough to occupy large share from that
nature, yet the outlook we develop upon reaching adulthood depends largely upon experiences from
the world we live in.

There are a lot of external agents we see right after we gain consciousness in the form of laws,
traditions, social and financial barriers, societal and family norms etc. and quickly we learn to abide by
those no matter how ridiculous they might seem. These forces shape our inner self that eventually
develop our behaviors. For instance an individual who spent most of his life in a society in which
throwing trash onto the streets is considered as unethical and reprehensible act might not follow the
suit laid down by the people of the region where the otherwise behavior is not considered as unethical.

Someone here can argue about the liberty of choice presented to every human being. Choice to select
from two extremes i.e. right or wrong, but that too is the effect of external forces we have experienced.
I would re-consider the above example. One might argue that a person is free to choose his actions
scrutinized by his/her conscience either to throw garbage on streets or to dispose them off properly.
Yes, for human being, we are blessed with the freedom of choice but that too not free from external
effects we have no control upon. A person among an irresponsible society can choose to act
responsible after witnessing the disadvantages and outcomes of being irresponsible.

Our experiences are actually the learning mechanism for our-selves upon which we develop our
behaviors. Same reply could satisfy the question about the two extreme behaviors developed by two
children of a drunk retard, who often uses violence upon their mother i.e. what do you expect from the
kid of such a retard to become. Without doubt liberty of choice played here an important role but that
too does not remain immune from the external experiences. In fact, the choice we make reflects our
behavior which is in turn the portrait carved by us on the canvas of our experiences and colored by our
ways to perceive those experiences.

In order to substantiate this further lets consider the outcomes of experiences or agents presents on
the two extreme ends of the spectrum. Many a times we come across examples in which we see good
individuals rise from darkest corners of the society. Clearly implications of external forces have
compelled an individual to think about altering his course of action with regards to other people around
him. Never once we have heard about the emergence of bad from the most enlightened circles in spite
of the fact that the door of liberty of choice remains open to everyone, every time.

In sum, our behavior is determined exclusively by the experiences we undergo that emerged from
traditions, laws and social, financial and ethnic restrictions. It is the process inherently attached to the
very nature of our selves and the surrounding in which we live and is conjured to shape our ways by
which we interact and relate to other people.

Argument Essay 8:

The following is a recommendation from the Board of Directors of Monarch Books.


We recommend that Monarch Books open a caf in its store. Monarch, having been in business at
the same location for more than twenty years, has a large customer base because it is known for its
wide selection of books on all subjects. Clearly, opening the caf would attract more customers. Space
could be made for the caf by discontinuing the children book section, which will probably become
less popular given that the most recent national census indicated a significant decline in the
percentage of the population under age ten. Opening a caf will allow Monarch to attract more
customers and better compete with Regal Books, which recently opened its own caf.

Write a response in which you discuss what questions would need to be answered in order to
decide whether the recommendation is likely to have the predicted result. Be sure to explain
how the answers to these questions would help to evaluate the recommendation.

The above authors recommendation to raise profit by opening a side business along an already well
running enterprise cannot be deemed as logically convincing, as it stands unless it offers appropriate
explanation to a number of questions.

Firstly, the author hasnt provided an explanation to this query that why he is considering opening a side
business provided a huge customer pool for wide spectrum of books. Usually entrepreneurs look out for
alternate option only when their ongoing endeavor does not seem to be thriving. For an already
lucrative enterprise, thinking about a side option could prove lethal. Because in doing so the people
running the business could lose focus to develop further their already running business.

Secondly, given the scenario one can ask a cogent question that was it the suggestion or advice, that the
Board of Directors of Monarch Books got from many of their customers or is it based on their own study
or survey. Every business setup demands a substantial amount of investment and one cannot base his
investment procedures on mere idea. Obviously a significant amount of evidence must be provided to
ensure that new investment endeavor will surely yield profit. No such study has been cited in authors
argument. So, it cannot be deem reliable.
Thirdly, author has recommended that the space for new enterprise could be sorted out by squeezing
the area donated to child book section. And In order to substantiate this recommendation he/ she cited
a national census which indicates that there has been a significant decline in childs population. This
takes us to conclude that the child population in the whole country has been declined at equal rate.
What if, few the states have experienced significant decrease in child population and the state in which
Monarch Books in situated hasnt been gone through any decline? Without having specific information
regarding the child population of the Monarch Books state we cannot assume decline in child
population on the result of general census report.

Lastly, the author hasnt mentioned the reason why their competitor has resorted to alternate business
endeavor. Maybe they are not achieving required profit margins through book selling alone? Or there
are no other cafs present in the vicinity of Regal Books so thats why Regal Books business developers
have thought to consider that having caf might boost their contemporary profit margin. Author also
needs to mention that whether or not any other caf is present in the vicinity of their outlet.

In sum, in order to evaluate authors recommendations efficiently I need to know the authors reason
and study conducted, if any, behind picking up an alternative enterprise while an already lucrative
business is in run. I also need a report or evidence of change in child population in that state and city in
which Monarch Books is situated. Furthermore, I need a proper explanation to why Regal Books have
started caf in its book store and the basis of customer response to their endeavor.
Issue Essay 14:

Governments should offer a free university education to any student who has been admitted to a
university but who cannot afford the tuition.

Write a response in which you discuss your views on the policy and explain your
reasoning for the position you take. In developing and supporting your position, you
should consider the possible consequences of implementing the policy and explain how
these consequences shape your position.

Education is not only the fundamental right but the need of every human being. Admittedly, making
education free of cost especially higher education would be an extremely noble virtue; however, policing
such a gesture would require extreme care and profound contemplation, for such legislation would be
likely to hurt self-respect of individual beings.

Devising a system to provide free higher education should be included in every governments top
priority programs. It will boost nations morale to opt for specialized degrees in their respective fields
providing organizations with specialized minds which would prove very fruitful for them. Obviously this
system would call for larger emphasis on merit. Furthermore, it can also play a substantial role in
keeping students focus during the course of their studies, keeping them from falling prey to distractions
often ubiquitous in university life, thus, providing a nation with a lot of educated class, who is more apt
in almost every sense.

Secondly, implementing such a policy would be based on the idea that education must reach every
individual of the society beyond social and economic barriers. Individuals would compete for the
opportunity to grow and develop, unfettered. Every economic strata of the society would want to play
their role in nations building because an air of equality would be built as a consequence to such a policy.
With an increase in number of private universities higher education is more becoming a lucrative
enterprise for investors, day by day eluding it for relatively penurious circle of the society. It is the
number of persons graduating from the quality higher educational institutes that determines the
nations resolve to develop and thrive. It is not the educated force that matters not the money
because without proper education and knowledge development might lose its trail and money would
become futile. Therefore, its the governments responsibility to strive for making free education
possible without burdening, especially the needy ones, with the liability of tuition fees.

On the contrary education comes at great price. Policing free education might put a tremendous strain
at the nations fiscal assets threatening the government planning to jeopardize. So, a keen effort is
required by the legislators to devise a sustainable system supporting free education. Moreover, a
considerable effort should be put in contemplating policing the free education that it must not be to
hurt the self-respect of students categorizing them as needy from the general mass.

In sum, to get educated is our right and to educate is our duty. As long as we stick to rule of making basic
human rights available to every human being free of cost, we will continue striving for making education
free of cost.

Argument Essay 9:
The following appeared in a memo from director of student housing at Buckingham College.

To serve the housing need of our students, Buckingham College should build a number of new
dormitories. Buckingham enrollment is growing and, based on current trends, will double by next 50
years, thus making existing dormitory space inadequate. Moreover, the average rent for an apartment
in our town has risen in recent years. Consequently, students will find it increasingly difficult to afford
off-campus housing. Finally, attractive new dormitories would make prospective students more likely
to enroll at Buckingham.

Write a response in which you discuss what specific evidence is needed to evaluate the
argument and explain how the evidence would weaken or strengthen the argument.

In this memo, the director of students residence at Buckingham College has highlighted the need for
new dormitories in the campuss hostel and based his recommendation upon a prescience that the
number of students would increase. I, however, consider this argument problematic in several respects.

First of all, the author hasnt provided any evidence of how the number of students would double? Is the
colleges management planning to increase the number of departments or courses offered at the
college? In case of no such plan, evidence must be provided to substantiate whether the colleges
management is planning to increase enrollments in existing departments. Without presenting any such
plans from the colleges management the authors recommendation cannot be deemed as required.

Secondly, nothing significant, like report from some reliable source or comprehensive survey is cited to
substantiate the surge in rental apartments rents. Maybe the director of housing has just asked only a
few students who are living outside the college premises, which is clearly not a representative of whole
students rental market. Even if we are to believe in such a rise in rents we cannot predict how far in
future these rises will last. It is entirely possible that this rise in rents of residence apartments is a mere
reaction of on-going inflation and will be controlled in future.
Furthermore, the author has made a claim that increased students enrollment will also call for increase
in dormitory number. This statement is not supported by any evidence. While this might be the case, the
director of housing at the Buckingham College ignores the possibility of increase in the number of
students commuting on daily basis from their respective homes. This scenario, if true, would render
deans argument for building new dormitory untenable.

In conclusion, the deans recommendation is not well supported. To strengthen it dean must provide
evidences related to colleges management plan about how they are anticipating the increase in the
number of students. An evidence related to increase in average rental rates of students apartment are
actually been increasing and this increase would continue in future and this effect would result in the
increased demand for dormitory housing.

Issue Essay 15:

Universities should require every student to take a variety of courses outside the students field of
study.

Write a response in which you discuss the extent to which you agree or disagree with
the claim. In developing and supporting your position, be sure to address the most
compelling reasons and/or examples that could be used to challenge your position.
No field is an island and it takes a tangible emphasis on studying different fields in order to become truly
educated. Studying different fields would help students to develop better understanding of their own
fields. While academicians could assist students decide which subjects to choose however, compulsion
to choose any of the subjects could prove detrimental to the very objective it entails.

Studying different courses especially those outside ones own field of study could be beneficial for
individual to better analyze and contribute more productively to their fields. For instance a better
understanding of pure and applied mathematics could be extremely helpful for a student thriving to
contemplate an obscure phenomenon of fluid dynamics. Technology is narrowing the gap between
people living in different parts of the world through different media enterprises. Problems originating
from the use these media, involves study of the variety of subjects i.e. social sciences and culture, law,
etc. A counter argument here could be developed based on that why a student should waste his/her
time pursuing those fields related very little to his own field. I would here say that the long term benefits
of this endeavor surely worth time consumption for a while.

To think out of the box and to widen ones horizon it is necessary for one to study subjects having an
overlapping realm with with his/her own field of study. Studying history, cultures of different countries,
economics, a bit of literature, management and administrative sciences, courses aimed to improve
interpersonal are all beneficial for the professional life of an Engineering or Science student. And for
people studying arts, culture, economics and literature could benefit from studying a tinge of scientific
method, problem solving in engineering, etc. To be able to contemplate on delicate overlapping regions
of different fields make a professional better in critical thinking and eventually transforms them into
good managers, directors, heads of departments and administrators.

It could also help polishing the enthusiasm and interest of an individual in a subject. For example if an
engineering student likes to write, he can choose to study literature during his graduation and can
become a part time writer, thus, having a side earning too. So, having been able to study outside ones
field not only brings out our hidden talent and gives a definite meaning to our innate predilections but
also grooms us to an extent that we could make a living out of it.

While, on one hand ability to choose freely from wide range of subjects to study could prove beneficial
to an individual in many ways, on the other hand it could also have many detrimental effects on
students. Number of subjects to be chosen at one time should be limited otherwise student could get
over loaded which would eventually harm his core subjects understanding. Students may be allowed to
choose any of the subjects after properly consulting with their teachers or professors and the latter
would need to act as their mentors who can have an in depth discussion with the students to help them
choose what is best for them to be a versatile professional at the same time holding them from going off
track from their core subjects.

In sum, it is tremendously beneficial for a student to study a variety of subjects. Obviously a special
attention is required to devise such a system to keep it from going haywire. However, one could even
argue and enlist the draw backs of such a system, in my opinion its benefits, for the students and the
society as a whole, outweighs its shortcomings.

Argument Essay 10:


Twenty years ago, Dr. Field, a noted anthropologist, visited the island of Tertia. Using an observation-
centered approach to studying Tertian culture, he concluded from his observations that children in
Tertia were reared by an entire village rather than by their own biological parents. Recently another
anthropologist, Dr. Karp, visited the group of islands that includes Tertia and used the interview-
centered method to study childrearing practices. In the interviews that Dr. Karp conducted with
children living in this group of island, the children spent much more time talking about their biological
parents than about other adults in the village. Dr. Karp decided that Dr. Fields conclusion about
Tertian village culture must be invalid. Some anthropologists recommend that to obtain accurate
information on Tertian child-rearing practices, future research on the subject should be conducted via
the interview-centered method.

Write a response in which you discuss what questions would need to be answered in
order to decide whether the recommendation and the argument on which it is based
are reasonable. Be sure to explain how the answers to these questions would help to
evaluate the recommendation.

In the above argument, Dr. Karps recommendation to adopt his research method, for it yields more
reliable results, is based on flawed premise. Furthermore, based on the above argument, Dr. Karps
outright rejection of Dr. Fields result is also unwarranted. Few questions need to be answered in order
to evaluate its logical standing.

Before getting bogged down into the discussion about which research method would yield most reliable
results, firstly, it is important here to note that the comparison upon which Dr. Karp has based his
recommendation could not be rendered as equitable. Comparing contemporary results with those made
surface two decades ago is probably not a cogent thing to do. As, the time transition could render huge
alterations in traditions and cultures. Therefore, one could not comment on the practicality and
fruitfulness of the methods employed, in different time frames, to study culture of an area. So, Dr. Karp
need to show that over the period of time Tertian peoples childrearing culture hasnt undergone any
change.

Secondly, Dr. Karp has included a number of islands in his study. There is no indication of whether he has
interviewed children of Island of Tertia or not. In such a situation Dr. Karps recommendation is
fundamentally flawed as his sample space does not contain elements to which he intends a comparison
with. Even if he has interviewed people and children of Island of Tertia, he needs to justify that he can
allot his generalized results to the people of Tertia as well by making it clear that the sample of people
he studied is actually a correct representor of the people of the island.

Thirdly, it is possible that there could be a different childrearing practice among different sectors of the
Island of Tertia and that Dr. Field has only made observation of those areas where childrearing was done
by whole sector/village. So, it is of significant importance for Dr. Karp mention to that his sample space
of children to be interviewed spread uniformly across the whole island.

Lastly, Dr. Karp rejected Dr. Fields result on the basis that he gathered children talking more about their
parents than any other adult. It is first of all, every childs natural inclination towards his/her biological
parents. And as a result of this natural inclination children usually talk more about their parents. The
reason that because children are not talking much about other adults, therefore, they are now only
being raised by their biological parents is insufficient. Dr. Karp needs to be more specific about the
questions he asked and the answers he got.
Moreover, it is not necessary for one method, if it is not delivering in one case, to be futile and deserving
to be rejected. On the contrary, if a method has worked in one case is not necessary that it will work and
yield similarly efficient results in all the other cases. So, Dr. Karp needs to answer that whether he is
rejecting Dr. Fields method due to some inherent fatal flaw or due to that it hasnt worked in just one
case that is too in his opinion.

In sum, Dr. Karp needs to provide evidence that Tertian Islands culture hasnt changed with regards to
childrearing over the past at least twenty five years. Moreover, is the sample he chosen for studying the
cultures on archipelagos, representative of the whole population, is uniformly distributed over the entire
individual islands and what kind of questions he qualified to conduct the study.

Issue Essay 16:

Some people believe that in order to be effective, political leaders must yield to public opinion and
abandon principles for the sake of compromise. Others believe that the most essential quality of an
effective leader is the ability to remain consistently committed to particular principles and objectives.

Write a response in which you discuss which view more closely aligns with your own
position and explain your reasoning for the position you take. In developing and
supporting your position, you should address both of the views presented.

A nations political philosophy is one of the most significant features in determining its path of
development. This philosophy is the reflection of political leaders will, ideals and principles. And it is the
political leaders that steer a nation through thick and thin utilizing their ideology. Therefore, an honest
politician knows the importance of clutching at his principles to everyones long term benefit. However,
at the same time a veteran politician also knows when and which of his principles to compromise for the
nations greater good.

What good could a political philosophy bring if it is vulnerable to changes and compromises? The values
and ideals upon which lies the very foundation of philosophies can never be subjected to change. So, the
political leader should never compromise his/her key beliefs no matter how tremendous the pressure
may be, because the founding values of an ideology if compromised, would eventually jeopardize the
cause the individual is standing for. For example, politician purporting a religious political ideology can
never pass or advocate legislation discordant with the religious believes or else he will lose his
credibility. In nary capacity it is meant that the political leaders must behave in a hardheaded or
opinionated manner.

Firm believes are founded on flexible grounds. They are the most generalized extracts that have
emerged from our life long experiences and knowledge. The more sound the more accommodating they
are. So, a wise and veteran politician could accommodate opinions without rendering hardly any harm
to his ideology. The matter of course, now, is of the opinions. Public opinions are usually spread across
wide spectrum; people hailing from diverse social, ethnic, economic and lingual backgrounds may
support entirely different causes. It is near to impossible to have such a wide range of opinions
accommodated into a political ideology.

Public opinions are often characterized as shortsighted and are generally, more vulnerable to bias,
prejudice and resentment. A smart politician should be able to work his/her way through pressure
rather than succumbing to it while without compromising to his/her values. If needed, to ease public
outcry, a democratically elected government can call in its armed forces to placate the unrest in some
volatile region, while some factions in public circles may be of the opinion for the government to resign.
On the other hand political leaders could suggest and implement a shift in foreign policy as a rejoinder to
some other nations aggressions discounting the possible public opinion to attack out rightly.

Obviously, there can be some circumstances which address no involvement of ones stringent principles
and values. These circumstances should be kept at distance and should not be allowed to indulge with
political leaders principles and ideology. Admittedly, it also depends upon ones interpretation of
circumstances but a wise politician has a better identification of the hair lines of his values. Out of
flawed policies, if the public is demanding resignation of a minister, then an honest political leader can
ask his minister to resign and it would not go in contradiction to his principles. In such instances, political
leader would not only be able to soothe the unrest but also could gain more popularity among the
masses.

In sum, for a veteran, honest and pragmatic political leader, public opinion, generally, would never come
in contradiction with his values, beliefs and ideology. He/ she must possess the shrewdness to work
his/her way through the time of crises or public outcry. Nevertheless, acting according to public opinion
in some petty cases would not only be considered as truthful but could prove to be fruitful as well.

Issue Essay 17:

Formal education tends to restrain our minds and spirits rather than set them free.

Write a response in which you discuss the extent to which you agree or disagree
with the statement and explain your reasoning for the position you take. In
developing and supporting your position, you should consider ways in which the
statement might or might not hold true and explain how these considerations shape
your position.
One of the primary functions of education is to render individuals the sense to act freely and
responsibly. It actually is education that enables human beings to remove shackles from theirs thoughts
and spirit and to prepare them to fight to get rid of them if such present physically. Nevertheless, if such
freedom left unchecked could become lethal for the very survival of society itself it is imperative to
make the process formal to reap maximum benefit from its practice.

Formalizing any process makes perpetual improvement in that process easy. Likewise formalizing
educational process would also tend to betterment of society and individuals in a long run. All the names
which have left an extraordinary mark in their respective fields were made able to do so by getting
formal education and we can render them the title of free thinkers. Someone may cite an example of
mavericks who had renounced the toil to formalize themselves; however, almost all have ended up help
building and rehabilitating the process of formal education. Why such a great achiever would surrender
himself to a system, he used to think could be of no help to him, obviously it is the sheer strength of
formal way, which could absorb anyone in itself to be raised to eminence, that had lured them to rethink
about their convictions.

Just think about if some process has no formal standing. Would it be wise to spread that as a trend for
everyone to emulate. Without doubt such kind of system would be devastating. To me formalizing
education means to set some renowned patterns which provide everyone an equal opportunity to work
for the betterment of them and for the society. We all know very well about the short coming of the
education system of the ancient days when only an opulent fraction of the society were allowed to
chase the endeavor of getting educated. Surely in such a scenario the ignorance of the indigent circles
would only get worse with the passage of time. Therefore, the formalization of the education has made
it possible for everyone to get education now. This is such a tremendous benefit that it would solely
overshadow any drawback rendered to formal education.

This is also possible that in some regions of the world formal education is reaping not as much good as it
was designed to reap. Maybe some people had got bogged down into too much formality that the
purpose of education has lost its meaning and the people who were thought to be free thinkers rather
becoming slaves in minds and spirits. Such a scenario is more likely to occur in those societies where
education is not in the approach of every individual. We can obviously overcome such conditions by
contemplating upon drawbacks of formal education and how to remove them. Admittedly, every society
has their own constraints so it is the job of thinking strata of the society to lead people to a responsible
redemption.

Formal education has one more striking benefit which is, only by formalizing the education elites of
every field have been able to come to a consensus about what has to be acknowledged in the realm of
knowledge and what has to be not. Had the formal education tend to shackle minds no one could have
come to such consensus. This is actually the best of formal education that we are now able to point to
new researches and study pathways for our future generations.

In sum, formal education has in many ways unleashed thinking rather than to shackle it altogether. It
could sometimes be exploited in ways to render constraints but that too could be solved by remaining in
the realm of formal education. Moreover, this process has produced many free thinkers in the past few
centuries that have inspired generations by their incredible achievements.

Argument Essay 11:

The council of Maple County, concerned about the countrys becoming overdeveloped, is debating a
proposed measure that would prevent the development of existing farmland in the county. But the
council is also concerned that such a restriction, by limiting the supply of new housing in the county.
Proponents of the measure note that Chestnut County established a similar measure ten years ago
and its housing prices have increased only modestly since. However, opponents of the measure note
that Pine County adopted restrictions on the development of new residential housing fifteen years
ago, and its housing prices have since more than doubled. The council currently predicts that the
proposed measure, if passed, will result in a significant increase in the housing prices in Maple County.

Write a response in which you discuss what questions would need to be answered in
order to decide whether the prediction and the argument on which it is based are
reasonable. Be sure to explain how the answers to these questions would help to
evaluate the prediction.

In order to evaluate the predictions of the Maple Countys council and the argument upon which it is
based one needs to carefully scrutinize the answers to a number of questions. I consider the above
foresight and reasoning of the councils members unconvincing and logically wanting.
Supply and demand of the product, item or a commodity has an inverse and direct relation, respectively,
with its price. The council needs to explain that how their decision to not to develop existing farms effect
the supply of land property to the prospect buyers. Whether an already adequately developed land is
present or not to build a society upon and secondly what is the status of demand, whether it is in the
residents best interests or not. An elaborate answer to above question would help us understand,
whether the prospect of developing new land would turn out as predicted or not. Surely with the land
development, the area of the land to be sold would be increased. If the council is not foreseeing an
increase in investment from the entrepreneurs in the construction industry then land development
would probably not result in increase in housing prices.

The proponents of the endeavor of land development in Maple County under what conditions had the
council of Chestnut County established the measures of development and exactly which area of the
county they developed and to what extent. Were those measures undertaken by Chestnut County the
very initial ones? Maybe the time to which the proponents are referring was that when the Chestnut
County had just started development. It is entirely possible that after the completion initial development
plan, ten years ago, Chestnut County hasnt seen any development thereafter. If such is the case then it
explains minute increase in property values in Chestnut County. Or else maybe they had developed little
societies with modest commercial prospect due to which over the past years, not much increase in
property value has been seen.

While the opponents have cited an example of Pine County where by adopting the policy of developing
new sites have resulted in doubling of the housing prices over the past fifteen years. This development is
pretty much intuitive. Span of fifteen years is enough to affect, significantly, the prices of land which
usually remain unstable. So, doubling of property price of Pine County does not very much reflects past
development measures of the countys council. Moreover, if the principal amount was not significantly
high the doubling of the property prices does not matter much. Therefore, the opponents need to show
that the change in the land prices of Pine County is significant and is directly related to development
projects within the county. Moreover, members opposing this development prospect need also to show
that how the land market of Maple County would emulate the results that had been seen in Pine
County. Because for different regions, it is possible, that the effects of a particular policy or actions
would yield different results.

In sum, to convince me on the predictions, members of council of Maple County need to present
contemporary market situation related to demand of such development projects and how much such an
endeavor will change the dynamics of available land for residential and commercial prospects?
Opponents and proponents need to show the relationship between the effects of development projects
on land prices, which were seen in Pine and Chestnut Counties, with the possible effects in Maple
County. Moreover, they also need to show contemporary market values and ongoing development
projects if it comes out that results seen in those two counties would be very likely to be emulated in
Maple County. Without appropriate answers to these questions, I would consider the above argument
unwarranted, as it stands.

Issue Essay 18:


Governments should focus on solving the immediate problems of today rather than on trying to solve
the anticipated problems of the future.

Write a response in which you discuss the extent to which you agree or
disagree with the recommendation and explain your reasoning for the
position you take. In developing and supporting your position, describe
specific circumstances in which adopting the recommendation would or
would not be advantageous and explain how these examples shape your
position.

Governments main goal is policy making, which is an art of anticipating long term problems and to
provide basis and guidelines to tackle them while remaining intact to nations ideology. A government is
generally, not supposed to solve contemporary issues in such a way that it would create a fuss for future
generations, however, some of the most pressing problems could be solved while neglecting their future
impacts.

Governments usually, foresee future problems while policing and suggest amendments when such arise.
For instance while making energy policy it is indispensable to state for how much time does the projects
could fulfil energy demand of the population increasing at such a rate. This act, obviously cater to future
demands and would suggest recommendations of how and when to act in order to avoid unpleasant
circumstances in the future. Admittedly there could arise some unforeseen circumstances due to some
policy implementation of the past but that too could be handled by interpreting and elaborating
generalized recommendations made in the policy.

Policy is always made to ripe long term benefits. And sometimes we have to sacrifice our current needs
and wishes for the future generations greater good. Maybe the government could start off by making
fewer than required schools just to make sure to not to compromise on education standard, in an area
where educational institutions were badly needed. Then later on in some specified time in future
government could go for actual requirement and also to some redundancies as well, after making sure
that all the educational institutions would reflect the same standard as displayed by the whole state or
country.

Knowing a rapidly changing world, what good a government can do if it is not capable of making policies,
laws so generalized, that contains in it solutions for the future situations. We live in a world in which our
every step yields consequences in future. How come governments be considered infallible to such
outcomes. Government policies are the ones whose consequences could affect many. So it is not
anything advisable to governments to not to pay heed to future situations. A government can deploy
army to placate agitation in some volatile area but need to consider future consequences of such an act
when army would be pulled out. Obviously policy makers foreseeing probable outcomes would never
advise to give a free hand to army to carry out operation.

Having said all above there could some circumstances arise, due to negligence of government in some
area or as a consequence of past policy. Admittedly immediate measure would have to be taken to
prevent such situation turning into the chaos but while doing so its aftermath must be taken into
account.
In sum it is almost always in every ones benefit to not to react impulsively merely on instincts but to give
considerable heed to consequences of our actions. In case of governments, to me not considering future
problems into their plans could prove disastrous for the nation.

Issue Essay 19:

Claim: The best way to understand the character of a society is to examine the character of the men
and women that the society chooses as its heroes or its role models.
Reason: Heroes and role models reveal a society's highest ideals.

Write a response in which you discuss the extent to which you agree or disagree with
the claim and the reason on which that claim is based.

Society is a vast term. Its an amalgam of people from almost every spectrum of life. Determining the
behavior of whom requires extreme meticulosity and is rather subjective than objective for it depends
upon persons considerations, who is doing research. Some of the ideals essential qualities could not
always be interpolated in order to understand the masses behavior. Over-generalization of an entitys
character could lead to flawed results. However, to have a better molded behavior, people need
someone to look up to and therefore, to some extent ideals do provide a frame work to carve out
societys characteristics.

Ideals and heroes are those few from a multitude who have no regard for their comfort and is due to
this very fact they became venerated for a society; on the contrary general public tend to remain in their
comfort zones. It should not be construed as people do not tend to follow their ideals but in actual they
do. The part debatable here is, to what extent people emulate their leaders. It is entirely possible for
two observers observing same set of features and reach at surprisingly opposite results for a society. For
instance, a society idolizing a religious figure comply with his teachings superficially with no heed paid to
the soul of why he has taught what he has taught. To an observer documenting their acts, for the sake of
determining whether or not are they following their leader, which they ostentatiously display to the
world, would come to a positive result that leaders character is enough to understand this societys
behavior. On the other hand some other researcher after noticing the societys moral downfall might
posit that this society hadnt understood the character of their leader and resorted to just show off their
false virtue. Therefore, there is a world of difference between peoples actions and their leaders
character.

As the heroes and role model reveals societys highest ideals how could one expect general public to
emulate those ideals. Sometimes people do not just tend to follow their heroes may be because they
consider them overly optimistic, rigorous or opinionated in some manner. They revere them as their
leaders but feel reluctant to follow their foot-steps. For instance, a societys hero had lead or is leading
his life in poverty struggling for the rights of his people; general public may not consider his foot-steps
worth it to be followed.

There could also some opinion difference interfere. Not necessarily people hold all the virtues of their
leader in high esteem. For some faction of the society some values could be emulated while others to be
left out. Or maybe people consider virtues of their leaders as outdated. For instance maybe in the past a
hero of a society had sought to take on violence in order for his voice to be heard but now with a boom
of technology and world converted to a global village, taking violent steps are not at all required for
merely making yourself audible.

However, in order to progress people always need someone to whom they look up to. They need some
inspiration to mobilize themselves and the society as a whole. Here, their heroes examples and
methodologies come into play and they tend to emulate their leaders teachings.

In sum one cannot understand a societys behavior by just studying the people whom these society hold
in high esteem for there are tremendously many factors affecting peoples behavior.
Issue Essay 20:
Governments should place few, if any, restrictions on scientific research and development.

Write a response in which you discuss the extent to which you agree or disagree
with the recommendation and explain your reasoning for the position you take. In
developing and supporting your position, describe specific circumstances in which
adopting the recommendation would or would not be advantageous and explain
how these examples shape your position.

Freedom is an essential requirement for any field of endeavor to flourish, be it sciences or any other.
The progress we see all around us would have never been possible if the pursuers of science had been
held under limitations. Given the destructive potential of many scientific fields it is however perilous to
let scientists alone in scrutinizing their deeds. This calls to the need for interference of political, social
and civil institutions. Obviously governments here have the responsibility to define and regulate the role
of scientific community, for nobody else has the power and infrastructure to embark on such a task.

We have seen in past that opinionated and dogmatic behavior of institutions having jurisdiction had
halted progress of many field of endeavors. Mostly we see instances of such bigotry during the reigns of
monarchs or in the societies of idolizing religious fanatics. Scientific community had worked hard not
only to attain professionalism and freed themselves from such fanatics but to raise thinking and
standards of societies. Now one cannot imagine a world where bigotry rules over rationale.

A far as scientific development and research is concerned we ourselves had posed many restriction over
it and most of them are merely the result of commonsense. Like we do not allow juveniles to get their
hands dirty with hazardous chemicals present in the chemistry lab or secondary or higher secondary
schools refrain from having practical researches involving radioactive elements. Any rational human
being will not oppose such restrictions as the cost to not involving students with dangerous scientific
things shall not matter as compared to an accident to which they are most vulnerable at that stage.

The first and foremost responsibility of the governments is to maintain the rule of law and to pay heed
to dissensions, to their policy, made within the limits set by the state. Many scientific researches could
be used to undermine states authority or to break law. Obviously, no government will allow researches
that could become potential threat to state itself. But that does not mean that government should halt
the progress in the fields that have a potential of becoming dangerous. Governments should make
institutions under its umbrella that propagate and reserve the right to do those researches. As the
confidence level on the government is certainly higher than isolated individuals or separate
organizations from keeping those results getting into the rogue hand or there misuse which have a
potential of becoming dangerous. It is include those filed as well in which there is a chance for an
individual to encroach other peoples privacy.

In sum, it sometimes become necessary for the government to restrict certain sectors of the societies in
pursuing certain fields due to the danger they posed if got into wrong hands. Nevertheless, this should
not mean that government would make these fields stagnant by halting research in them. In that case it
is the responsibility of government to make arrangements for these researches to go on under its own
umbrella.

Argument Essay 12:

Fifteen years ago, Omega University implemented a new procedure that encouraged students to
evaluate the teaching effectiveness of all their professors. Since that time, Omega professors have
begun to assign higher grades in their classes, and overall student grade averages at Omega have risen
by 30 percent. Potential employers, looking at this dramatic rise in grades, believe that grades at
Omega are inflated and do not accurately reflect student achievement; as a result, Omega graduates
have not been as successful at getting jobs as have graduates from nearby Alpha University. To enable
its graduates to secure better jobs, Omega University should terminate student evaluation of
professors.

Write a response in which you discuss what specific evidence is needed to evaluate
the argument and explain how the evidence would weaken or strengthen the
argument.

The speaker in the above argument has made some unwarranted assumptions and linked apparently
different scenarios without presenting any evidence to support that connection. With the lack of
evidence the speaker fails to make a cogent case and I dont consider his conclusion and the reasoning
upon which it is based persuasive, as it stands.

First of all, the speaker has linked a long held practice of teachers evaluation to decrease in acceptation
of students from Omega University to potential employers. Despite of this being a quiet daunting task,
speaker should provide evidence to support this linkage. For instance, a documented proof, regarding
teachers, would be required that establishes that after incorporating teachers evaluation into practice,
their teaching standards got dilapidated and also with that their grading criteria got relaxed. The proof
could include a number of factors like average amount of teaching time a particular teacher gives to his
class before and after the implementation of teachers evaluation or alteration in views of the students
about their teachers and their teaching methods over that period of time.

Secondly, it is also not been firmly established that how could the employers conclude that the grade of
students of Omega University do not depict actual students capabilities or academic prowess. Do the
employers give some kind of test in which they have seen a considerable decline in performances of
students from Omega University? If such is the case, speaker need to provide test statistics over the
years that clearly demonstrate that students performance has been declined over the years in those
tests. Even if the Omega graduates performance in the test given by the employers is declining, the
speaker needs to show that teachers selection criteria of the university has remained the same over the
years, in fact got even rigorous as compared to what was 15 years before. Teachers relations to
universitys administration are going well, teachers are getting proper raises at proper time and
students selection criteria has remain the same over the years. If all is going well then a decline in
teaching standards and Universitys rigorous academic system can be attributed to teachers evaluation
system.
Thirdly, the speaker need to show that there is no compulsion from the Universitys administration that
dictates teachers to pass all or most of the students and in order to comply with that requirement
teacher are giving easy tests. Moreover it is also entirely possible that Alpha University admits more
sharp and intelligent students which after going through universitys rigorous academic system are
sharpen to an extreme level and therefore, locking a major share for them in job market. Speaker needs
to show that both the universities have the same admission criteria and system for students to get
admissions and also both the universities have same caliber of teachers.

In the final analysis, I would require comprehensive and quantitative proof of declining in teaching
standards of the teachers in the Omega University after the implementation of the evaluation criteria.
Furthermore, I would also require an evidence demonstrating that teachers of the Omega University are
mentally happy with their jobs, are equally experienced and qualified as compared to the teachers of
other universities, students admission criteria has the same factor of rigor as the other universities have
and that the university has continually upgraded its course content with the changing and flourishing
need of the industry. Without these evidences I would regard the authors above reasoning flawed
logically unconvincing, as it stands.

Issue Essay 21:

The best way to teach is to praise positive actions and to ignore negative ones.

Write a response in which you discuss the extent to which you agree or disagree
with the recommendation and explain your reasoning for the position you take. In
developing and supporting your position, describe specific circumstances in which
adopting the recommendation would or would not be advantageous and explain
how these examples shape your position.

Teaching is the multi-dimensional art. It is the way to make an individual the best he/she can and also it
is the one of the most reliable method to identify the best among the masses. In the above statement
the author while asserting his view related to the best way of teaching ignores many important factors.

While it may be true that by lauding positive actions could inspire an individual to further strive for
betterment but at the same time, this laudation when tied with disregard of negative deeds may
produce unwanted results. For instance a teachers praise for a students uncanny ability to grasp
mathematical problems may inspire him to further exploit his abilities to broaden his arena of
mathematical prowess however, if at the same time the teacher does not pay heed to the same
students careless behavior of not attending his lectures regularly, out of having an over confidence
upon his abilities. This teachers behavior shall prove to be more harmful as compared to the good he is
inducing and the student would not be able to become what he can at best.

Secondly, effective teachers know that their teaching style needs to be varied from student to student
and from situation to situation for a same student. For a prodigious student in the class an honest
teacher may resort to pointing weak sides more than pointing at the bright sides of his talent in order to
achieve a greater good for that student. While may be for a relatively weaker student of the class the
teacher more frequently commend for his positive show of intent for keeping him motivated until that
motivation becomes a part of his nature. Here the teacher has to maintain a certain balance between
pointing negative aspects of talented ones, for keeping the teachers critical views going to such an
extent that prodigious goes prodigal.
As, said before that teaching is a multi-dimensional art. Praising and ignoring various aspects of the
students characters and deed may be an effective way of teaching but it is not an only effective way.
There are other ways also that could produce best results. After employing every technique, a teacher
could ignore an intelligent but careless student so that he may start giving serious attention to his
studies.

In the final analysis, there is no one that could be regarded as the best way. A best teacher must learn to
improvise his methods according to the situations. Secondly whether to praise and what to praise is a
highly subjective topic and could be changed from situation to situation, same goes for ignoring as well.

You might also like